Über das relativistische Wagengleichzeitigkeitsproblem [geschlossen]

Ich bin völlig verwirrt mit diesem Problem. Die Situation besteht im Grunde darin, zu wissen, welche Ereignisse in einem Frame aufgenommen werden müssen, um sie in einem anderen synchronisiert zu sehen. Ich habe die Situation so skizziert:

Wagen

Es gibt zwei Wagen A und B in Ruhelänge sich mit Geschwindigkeit bewegen v Und v aus Sicht eines Beobachters C. Im Waggon B gibt es eine Lampe, die regelmäßig blinkt.

Angenommen (das ist eigentlich die Metapher des ursprünglichen Problems), dies ist ein relativistischer Tanzwettbewerb, bei dem C der Richter ist und die Beobachter bei A und B ein tanzendes Paar sind. Was ich wissen möchte, ist, wann die Beobachter bei A und B nach einem Blinzeln tanzen sollten, damit der Richter C ihren Tanz synchronisiert sieht.

Was ich versuche ist, die Ereignisse ihrer individuellen Reaktion der Betrachter am Rahmen der Lampe zu sehen und sie dann auf den Rahmen der Jury zu transformieren. Also lassen ( T B , X B , 0 , 0 ) das Ereignis sein, wenn der Tänzer bei B reagiert und ( T A , X A , 0 , 0 ) das Ereignis, wenn der Tänzer bei A vom Lampenrahmen aus gesehen reagiert, im Juryrahmen,

T B ' = γ v ( T B v X B C 2 ) T A ' = γ v ( T A + v X A C 2 )
die gleich sein müssen, damit C diese Ereignisse als gleichzeitig ansieht, dann (als γ v = γ v )
T B v X B C 2 = T A + v X A C 2
von der Lampe aus gesehen, X B = immer so
T A = T B v C 2 v X A C 2
und ab hier glaube ich nicht mehr weiter zu wissen; X A sollte mindestens gleich sein , also lasse ich X A = X a für einige X a > 0 was ich hoffe feststellen zu können, daher
T A = T B v X a C 2
Wenn ich lasse X a = 0 , Woher T A ' = T B ' impliziert T A = T B ? Was soll ich damit machen X a ? Ich denke auch ans Vermieten T B = C wäre schön, da dies die Zeit ist, die das Licht braucht, um den Beobachter in B zu erreichen
T A = C v X a C 2
aber andererseits weiß ich nicht, was ich damit anfangen soll X a . Ich dachte auch daran, drei Referenzrahmen zu nehmen, zwei für die Waggons und einen für C, aber ich kam nicht weiter. Ich dachte auch, dass die Geschwindigkeitsaddition oder die Zeitdilatation im Weg stehen sollte, aber ich kann nicht sehen, wo. Wie soll also mit diesem Problem umgegangen werden und mit welchen Konzepten?

Antworten (1)

Ich denke nicht, dass das von Ihnen verwendete Beispiel gut ist, um die Relativität der Gleichzeitigkeit zu veranschaulichen. Es hat die zusätzliche Komplikation, dass 3 Referenzrahmen verwendet werden, wenn zwei ausreichen sollten. Ich konnte Ihrer Herleitung nicht folgen, aber hier ist eine viel einfachere und leichter zu verstehende.

Stellen Sie sich zwei Beobachter vor, einen in der Mitte eines rasenden Waggons und einen anderen Beobachter, der auf einem Bahnsteig steht, während der Zug vorbeifährt.
Ein Lichtblitz wird in der Mitte des Waggons abgegeben, während die beiden Beobachter aneinander vorbeigehen. Der Beobachter an Bord des Zuges sieht die Vorder- und Rückseite des Waggons in festen Abständen von der Lichtquelle, und daher wird das Licht laut diesem Beobachter die Vorder- und Rückseite des Waggons gleichzeitig erreichen. Der auf dem Bahnsteig stehende Beobachter hingegen sieht, wie sich das Heck des Waggons auf die Stelle zubewegt (aufholt), an der der Blitz abgegeben wurde, und die Front des Waggons sich davon wegbewegt. Da die Lichtgeschwindigkeit endlich und für alle Beobachter in allen Richtungen gleich ist, muss das Licht, das auf die Rückseite des Zuges gerichtet ist, eine geringere Entfernung zurücklegen als das Licht, das auf die Vorderseite gerichtet ist. Somit treffen die Lichtblitze zu unterschiedlichen Zeiten auf die Enden des Waggons.

1) aus Sicht der Zugperson erreicht das Licht beide Enden gleichzeitig Geben Sie hier die Bildbeschreibung eine

2) Aus Sicht der Person auf dem Bahnsteig erreicht das Licht zuerst das Ende des Zuges

Geben Sie hier die Bildbeschreibung ein

Vielen Dank für Ihre Klarstellung dort physical.stackexchange.com/questions/267691/… . Ich wünschte, ich hätte das Bild früher gesehen, um meine Frage klarer zu stellen. Was ich immer noch nicht verstehe, ist, dass sich der Zug in Frame 1) relativ zum Licht bewegt (da die Lichtgeschwindigkeit konstant ist und nicht durch den Referenzrahmen geändert wird), daher sollte das Licht die gleiche Zeit benötigen, um es zu erreichen das vordere Ende in beiden 1) & 2)